MedSurge3 FINAL

Ace your homework & exams now with Quizwiz!

A...B...C...D...E... OF triage (primary survey)

A = airway & cervical spine immobilized B = breathing (admin O2) C = circulation/control hemorrhage (check carotid & femoral pulse) D = disability (LOC, ROM, sensation) E = expose (take pt's clothes off for full exam)

Bladder bundle for prevention of HAUTI

A = aseptic insertion B = bladder US C = condom or intermittent catheterization D = do NOT use indwelling cath unless you must E = early removal of catheters

When preparing to cool a patient who is to begin therapeutic hypothermia, which intervention will the nurse plan to do (select all that apply)? a. Assist with endotracheal intubation. b. Insert an indwelling urinary catheter. c. Begin continuous cardiac monitoring. d. Obtain an order to restrain the patient. e. Prepare to give sympathomimetic drugs.

A, B, C -assist w/ endotracheal intubation -insert indwelling urinary catheter -begin cont cardiac monitor

Which information will the nurse consider when deciding what nursing actions to delegate to a licensed practical/vocational nurse (LPN/LVN) who is working on a medical-surgical unit (select all that apply)? a. Institutional policies b. Stability of the patient c. State nurse practice act d. LPN/LVN teaching abilities e. Experience of the LPN/LVN

A, B, C, E -institutional policies -stability of the pt -state nurse practice act -experience of LPN/LVN

A patient with Parkinsons disease is admitted to the hospital for treatment of pneumonia. Which nursing interventions will be included in the plan of care (select all that apply)? a. Use an elevated toilet seat. b. Cut patients food into small pieces. c. Provide high-protein foods at each meal. d. Place an armchair at the patients bedside. e. Observe for sudden exacerbation of symptoms.

A, B, D -use an elevated toilet seat -cut pt's food into small pieces -place an armchair at the pt's bedside

When caring for a patient who experienced a T2 spinal cord transection 24 hours ago, which collaborative and nursing actions will the nurse include in the plan of care (select all that apply)? a. Urinary catheter care b. Nasogastric (NG) tube feeding c. Continuous cardiac monitoring d. Maintain a warm room temperature e. Administration of H2 receptor blockers

A,C,D,E -urinary catheter care -continous cardiac monitoring -maintain warm room temperature -admin H2 receptor blockers

The nurse provides discharge instructions for a 40-yr-old woman newly diagnosed with cardiomyopathy. Which statement indicates that further teaching is necessary? a. "I will avoid lifting heavy objects." b. "I can drink alcohol in moderation." c. "My family will need to take a CPR course." d. "I will reduce stress by learning guided imagery."

B "I can drink alcohol in moderation." Patients with cardiomyopathy should avoid alcohol consumption, especially in patients with alcohol-related dilated cardiomyopathy. Avoiding heavy lifting and stress, as well as family members learning CPR, are recommended teaching points.

The emergency department (ED) nurse is initiating therapeutic hypothermia in a patient who has been resuscitated after a cardiac arrest. Which actions in the hypothermia protocol can be delegated to an experienced licensed practical/vocational nurse (LPN/LVN) (select all that apply)? a. Continuously monitor heart rhythm. b. Check neurologic status every 2 hours. c. Place cooling blankets above and below patient. d. Give acetaminophen (Tylenol) 650 mg per nasogastric tube. e. Insert rectal temperature probe and attach to cooling blanket control panel.

C, D, E -place cooling pt above & below pt -give acetaminophen/Tylenol 650mg per NGT -insert rectal temp probe & attach to cooling blanket control panel

3 F's & H of triage (secondary survey)

F = full set of VS F = family presence F = five interventions (EKG, spO2, Foley, NGT, labs) H = hx & H2T

A patient has ST segment changes that support an acute inferior wall myocardial infarction. Which lead would be best for monitoring the patient?

II

Active TB Meds

RIPE (rifampin, isoniazid, pyrazinamide, ethambutol)

The nurse has received the laboratory results for a patient who developed chest pain 4 hours ago and may be having a myocardial infarction. The most important laboratory result to review will be...

Troponins T and I

Which factors will the nurse consider when calculating the CURB-65 score for a patient with pneumonia (select all that apply)? a. Age b. Blood pressure c. Respiratory rate d. Oxygen saturation e. Presence of confusion f. Blood urea nitrogen (BUN) level

a, b, c, e, f -age -BP -RR -presence of confusion -BUN level

Following an earthquake, patients are triaged by emergency medical personnel and are transported to the emergency department (ED). Which patient will the nurse need to assess first? a. A patient with a red tag b. A patient with a blue tag c. A patient with a black tag d. A patient with a yellow tag

a. A patient with a red tag

A 64-year-old patient who has amyotrophic lateral sclerosis (ALS) is hospitalized with pneumonia. Which nursing action will be included in the plan of care? a. Assist with active range of motion (ROM). b. Observe for agitation and paranoia. c. Give muscle relaxants as needed to reduce spasms. d. Use simple words and phrases to explain procedures.

a. Assist with active range of motion (ROM). (ALS causes progressive muscle weakness, but assisting the patient to perform active ROM will help maintain strength as long as possible)

Which action will the nurse include in the plan of care for a 72-year-old woman admitted with multiple myeloma? a. Monitor fluid intake and output. b. Administer calcium supplements. c. Assess lymph nodes for enlargement. d. Limit weight bearing and ambulation.

a. Monitor fluid intake and output (A high fluid intake and urine output helps prevent the complications of kidney stones caused by hypercalcemia and renal failure caused by deposition of Bence-Jones protein in the renal tubules.)

Which nursing action for a patient with chronic obstructive pulmonary disease (COPD) could the nurse delegate to experienced unlicensed assistive personnel (UAP)? a. Obtain oxygen saturation using pulse oximetry. b. Monitor for increased oxygen need with exercise. c. Teach the patient about safe use of oxygen at home. d. Adjust oxygen to keep saturation in prescribed parameters.

a. Obtain oxygen saturation using pulse oximetry.

A hospitalized 31-year-old patient with a history of cluster headache awakens during the night with a severe stabbing headache. Which action should the nurse take first? a. Start the ordered PRN oxygen at 6 L/min. b. Put a moist hot pack on the patients neck. c. Give the ordered PRN acetaminophen (Tylenol). d. Notify the patients health care provider immediately.

a. Start the ordered PRN oxygen at 6 L/min. (Acute treatment for cluster headache is administration of 100% oxygen at 6 to 8 L/min. If the patient obtains relief with the oxygen, there is no immediate need to notify the health care provider. Cluster headaches last only 60 to 90 minutes, so oral pain medications have minimal effect. Hot packs are helpful for tension headaches but are not as likely to reduce pain associated with a cluster headache.)

Which intervention by a new nurse who is caring for a patient who has just had an implantable cardioverter-defibrillator (ICD) inserted indicates a need for more education about care of patients with ICDs? a. The nurse assists the patient to do active range of motion exercises for all extremities. b. The nurse assists the patient to fill out the application for obtaining a Medic Alert ID. c. The nurse gives amiodarone (Cordarone) to the patient without first consulting with the health care provider. d. The nurse teaches the patient that sexual activity usually can be resumed once the surgical incision is healed.

a. The nurse assists the patient to do active range of motion exercises for all extremities.

A patient has a new order for magnetic resonance imaging (MRI) to evaluate for left femur osteomyelitis after a hip replacement surgery. Which information indicates that the nurse should consult with the health care provider before scheduling the MRI? a. The patient has a pacemaker. b. The patient is claustrophobic. c. The patient wears a hearing aid. d. The patient is allergic to shellfish.

a. The patient has a pacemaker.

The urgent care center protocol for tick bites includes the following actions. Which action will the nurse take first when caring for a patient with a tick bite? a. Use tweezers to remove any remaining ticks. b. Check the vital signs, including temperature. c. Give doxycycline (Vibramycin) 100 mg orally. d. Obtain information about recent outdoor activities.

a. Use tweezers to remove any remaining ticks.

The nurse advises a patient with myasthenia gravis (MG) to a. perform physically demanding activities early in the day. b. anticipate the need for weekly plasmapheresis treatments. c. do frequent weight-bearing exercise to prevent muscle atrophy. d. protect the extremities from injury due to poor sensory perception.

a. perform physically demanding activities early in the day.

Which nursing action has the highest priority for a patient who was admitted 16 hours previously with a C5 spinal cord injury? a. Cardiac monitoring for bradycardia b. Assessment of respiratory rate and effort c. Application of pneumatic compression devices to legs d. Administration of methylprednisolone (Solu-Medrol) infusion

b. Assessment of respiratory rate and effort

Several patients have been hospitalized for diagnosis of neurologic problems. Which patient will the nurse assess first? a. Patient with a transient ischemic attack (TIA) returning from carotid duplex studies b. Patient with a brain tumor who has just arrived on the unit after a cerebral angiogram c. Patient with a seizure disorder who has just completed an electroencephalogram (EEG) d. Patient prepared for a lumbar puncture whose health care provider is waiting for assistance

b. Patient with a brain tumor who has just arrived on the unit after a cerebral angiogram (cerebral angiogram req catheter insertion in femoral artery bleeding is a potential risk that must be monitored)

The nurse is providing education to nursing staff on quality care initiatives. Which statement would be the most accurate description of the impact of health care financing on quality care? a. Hospitals are reimbursed for all costs incurred if care is documented electronically. b. Payment for patient care is primarily based on clinical outcomes and patient satisfaction. c. If a patient develops a catheter-related infection, the hospital receives additional funding. d. Because hospitals are accountable for overall care, it is not nursings responsibility to monitor care delivered by others.

b. Payment for patient care is primarily based on clinical outcomes and patient satisfaction.

Which information will the nurse include when teaching the patient with a urinary tract infection (UTI) about the use of phenazopyridine (Pyridium)? a. Pyridium may cause photosensitivity b. Pyridium may change the urine color. c. Take the Pyridium for at least 7 days. d. Take Pyridium before sexual intercourse.

b. Pyridium may change the urine color.

The nurse plans health care for a community with a large number of recent immigrants from Vietnam. Which intervention is the most important for the nurse to implement? a. Hepatitis testing b. Tuberculosis screening c. Contraceptive teaching d. Colonoscopy information

b. Tuberculosis screening (TB incidence higher in immigrants from Vietnam)

The nurse is planning a teaching session with a patient newly diagnosed with migraine headaches. To assess a patients readiness to learn, which question should the nurse ask? a. What kind of work and leisure activities do you do? b. What information do you think you need right now? c. Can you describe the types of activities that help you learn new information? d. Do you have any religious beliefs that are inconsistent with the planned treatment?

b. What information do you think you need right now?

The nurse will explain to the patient who has a T2 spinal cord transection injury that a. use of the shoulders will be limited. b. function of both arms should be retained. c. total loss of respiratory function may occur. d. tachycardia is common with this type of injury.

b. function of both arms should be retained.

A 22-year-old female patient seen in the clinic for a bladder infection describes the following symptoms. Which information is most important for the nurse to report to the health care provider? a. Urinary urgency b. Left-sided flank pain c. Intermittent hematuria d. Burning with urination

b. left-sided flank pain (Flank pain indicates that the patient may have developed pyelonephritis as a complication of the bladder infection. The other clinical manifestations are consistent with a lower urinary tract infection (UTI).

A patient who is unconscious after a fall from a ladder is transported to the emergency department by emergency medical personnel. During the primary survey of the patient, the nurse should a. obtain a complete set of vital signs. b. obtain a Glasgow Coma Scale score. c. ask about chronic medical conditions. d. attach a cardiac electrocardiogram monitor.

b. obtain a Glasgow Coma Scale score.

It is most important that the nurse ask a patient admitted with acute glomerulonephritis about a. history of kidney stones. b. recent sore throat and fever. c. history of high blood pressure. d. frequency of bladder infections.

b. recent sore throat and fever. (Acute glomerulonephritis frequently occurs after a streptococcal infection such as strep throat.)

A female patient with a suspected urinary tract infection (UTI) is to provide a clean-catch urine specimen for culture and sensitivity testing. To obtain the specimen, the nurse will a. have the patient empty the bladder completely, then obtain the next urine specimen that the patient is able to void. b. teach the patient to clean the urethral area, void a small amount into the toilet, and then void into a sterile specimen cup. c. insert a short sterile mini catheter attached to a collecting container into the urethra and bladder to obtain the specimen. d. clean the area around the meatus with a povidone-iodine (Betadine) swab, and then have the patient void into a sterile container.

b. teach the patient to clean the urethral area, void a small amount into the toilet, and then void into a sterile specimen cup.

The nurse assessing a 54-year-old female patient with newly diagnosed trigeminal neuralgia will ask the patient about a. visual problems caused by ptosis. b. triggers leading to facial discomfort. c. poor appetite caused by loss of taste. d. weakness on the affected side of the face.

b. triggers leading to facial discomfort.

A male patient in the clinic provides a urine sample that is red-orange in color. Which action should the nurse take first? a. Notify the patients health care provider. b. Teach correct midstream urine collection. c. Ask the patient about current medications. d. Question the patient about urinary tract infection (UTI) risk factors.

c. Ask the patient about current medications.

The nurse obtains the following data when assessing a patient who experienced an ST-segment-elevation myocardial infarction (STEMI) 2 days previously. Which information is most important to report to the health care provider? a. The troponin level is elevated. b. The patient denies ever having a heart attack. c. Bilateral crackles are auscultated in the mid-lower lobes. d. The patient has occasional premature atrial contractions (PACs).

c. Bilateral crackles are auscultated in the mid-lower lobes. (possible HF developing)

A 27-year-old patient tells the nurse that she would like a prescription for oral contraceptives to control her premenstrual dysphoric disorder (PMD-D) symptoms. Which patient information is most important to communicate to the health care provider? a. Bilateral breast tenderness b. Frequent abdominal bloating c. History of migraine headaches d. Previous spontaneous abortion

c. History of migraine headaches

A patient in metabolic alkalosis is admitted to the emergency department, and pulse oximetry (SpO2) indicates that the O2 saturation is 94%. Which action should the nurse take next? a. Administer bicarbonate. b. Complete a head-to-toe assessment. c. Place the patient on high-flow oxygen. d. Obtain repeat arterial blood gases (ABGs).

c. Place the patient on high-flow oxygen. (shift to left: less O2 in tissues)

The health care provider writes an order for bacteriologic testing for a patient who has a positive tuberculosis skin test. Which action should the nurse take? a. Teach about the reason for the blood tests. b. Schedule an appointment for a chest x-ray. c. Teach about the need to get sputum specimens for 2 to 3 consecutive days. d. Instruct the patient to expectorate three specimens as soon as possible.

c. Teach about the need to get sputum specimens for 2 to 3 consecutive days.

A patient returned from a laparoscopic Nissen fundoplication for hiatal hernia 4 hours ago. Which assessment finding is most important for the nurse to address immediately? a. The patient is experiencing intermittent waves of nausea. b. The patient complains of 7/10 (0 to 10 scale) abdominal pain. c. The patient has absent breath sounds in the left anterior chest. d. The patient has hypoactive bowel sounds in all four quadrants.

c. The patient has absent breath sounds in the left anterior chest. (may indicate pneumothorax)

24. Which information from a patient who had a transurethral resection with fulguration for bladder cancer 3 days ago is most important to report to the health care provider? a. The patient is voiding every 4 hours. b. The patient is using opioids for pain. c. The patient has seen clots in the urine. d. The patient is anxious about the cancer.

c. The patient has seen clots in the urine.

A patient is being evaluated for Alzheimers disease (AD). The nurse explains to the patients adult children that a. the most important risk factor for AD is a family history of the disorder. b. new drugs have been shown to reverse AD dramatically in some patients. c. a diagnosis of AD is made only after other causes of dementia are ruled out. d. the presence of brain atrophy detected by magnetic resonance imaging (MRI) will confirm the diagnosis of AD.

c. a diagnosis of AD is made only after other causes of dementia are ruled out.

A 27-year-old patient is hospitalized with new onset of Guillain-Barr syndrome. The most essential assessment for the nurse to carry out is a. determining level of consciousness. b. checking strength of the extremities. c. observing respiratory rate and effort. d. monitoring the cardiac rate and rhythm.

c. observing respiratory rate and effort. (most serious complication is respiratory failure)

A patient who had a C7 spinal cord injury a week ago has a weak cough effort and audible rhonchi. The initial intervention by the nurse should be to a. administer humidified oxygen by mask. b. suction the patients mouth and nasopharynx. c. push upward on the epigastric area as the patient coughs. d. encourage incentive spirometry every 2 hours during the day.

c. push upward on the epigastric area as the patient coughs.

When assessing a patient who has just arrived after an automobile accident, the emergency department nurse notes tachycardia and absent breath sounds over the right lung. For which intervention will the nurse prepare the patient? a. Emergency pericardiocentesis b. Stabilization of the chest wall with tape c. Administration of an inhaled bronchodilator d. Insertion of a chest tube with a chest drainage system

d. Insertion of a chest tube with a chest drainage system

The nurse assesses an older patient who takes diuretics and has a possible urinary tract infection (UTI). Which action should the nurse take first? a. Palpate over the suprapubic area. b. Inspect for abdominal distention. c. Question the patient about hematuria. d. Invite the patient to use the bathroom.

d. Invite the patient to use the bathroom. (Before beginning the assessment of an older patient with a UTI and on diuretics, the nurse should have the patient empty the bladder because bladder fullness or discomfort will distract from the patients ability to provide accurate information. The patient may seem disoriented if distracted by pain or urgency. The physical assessment data are obtained after the patient is as comfortable as possible.)

The nurse admits a patient who has a diagnosis of an acute asthma attack. Which statement indicates that the patient may need teaching regarding medication use? a. I have not had any acute asthma attacks during the last year. b. I became short of breath an hour before coming to the hospital. c. Ive been taking Tylenol 650 mg every 6 hours for chest-wall pain. d. Ive been using my albuterol inhaler more frequently over the last 4 days.

d. Ive been using my albuterol inhaler more frequently over the last 4 days. (The increased need for a rapid-acting bronchodilator should alert the patient that an acute attack may be imminent and that a change in therapy may be needed.)

When rewarming a patient who arrived in the emergency department (ED) with a temperature of 87 F (30.6 C), which assessment indicates that the nurse should discontinue active rewarming? a. The patient begins to shiver. b. The BP decreases to 86/42 mm Hg. c. The patient develops atrial fibrillation. d. The core temperature is 94 F (34.4 C).

d. The core temperature is 94 F (34.4 C).

The nurse admits a patient to the hospital and develops a plan of care. What components should the nurse include in the nursing diagnosis statement? a. The problem and the suggested patient goals or outcomes b. The problem with possible causes and the planned interventions c. The problem, its cause, and objective data that support the problem d. The problem with an etiology and the signs and symptoms of the problem

d. The problem with an etiology and the signs and symptoms of the problem

The nurse supervises unlicensed assistive personnel (UAP) who are providing care for a patient with right lower lobe pneumonia. The nurse should intervene if which action by UAP is observed? a. UAP splint the patients chest during coughing. b. UAP assist the patient to ambulate to the bathroom. c. UAP help the patient to a bedside chair for meals. d. UAP lower the head of the patients bed to 15 degrees.

d. UAP lower the head of the patients bed to 15 degrees.

A 76-year-old patient is being treated with carbidopa/levodopa (Sinemet) for Parkinsons disease. Which information is most important for the nurse to report to the health care provider? a. Shuffling gait b. Tremor at rest c. Cogwheel rigidity of limbs d. Uncontrolled head movement

d. Uncontrolled head movement (Dyskinesia is an adverse effect of the Sinemet, indicating a need for a change in medication or decrease in dose. The other findings are typical with Parkinsons disease)

The nurse develops a teaching plan to help increase activity tolerance at home for an older adult with severe chronic obstructive pulmonary disease (COPD). Which instructions would be most appropriate for the nurse to include in the plan of care? a. Stop exercising when short of breath. b. Walk until pulse rate exceeds 130 beats/minute. c. Limit exercise to activities of daily living (ADLs). d. Walk 15 to 20 minutes daily at least 3 times/week.

d. Walk 15 to 20 minutes daily at least 3 times/week.

A 72-year-old female patient is brought to the clinic by the patients spouse, who reports that she is unable to solve common problems around the house. To obtain information about the patients current mental status, which question should the nurse ask the patient? a. Are you sad? b. How is your self-image? c. Where were you were born? d. What did you eat for breakfast?

d. What did you eat for breakfast?

Following an acute myocardial infarction, a previously healthy 63-year-old develops clinical manifestations of heart failure. The nurse anticipates discharge teaching will include information about a. digitalis preparations. b. b-adrenergic blockers. c. calcium channel blockers. d. angiotensin-converting enzyme (ACE) inhibitors.

d. angiotensin-converting enzyme (ACE) inhibitors.

15. While caring for a 23-year-old patient with mitral valve prolapse (MVP) without valvular regurgitation, the nurse determines that discharge teaching has been effective when the patient states that it will be necessary to a. take antibiotics before any dental appointments. b. limit physical activity to avoid stressing the heart. c. take an aspirin a day to prevent clots from forming on the valve. d. avoid use of over-the-counter (OTC) medications that contain stimulant drugs.

d. avoid use of over-the-counter (OTC) medications that contain stimulant drugs. (can exacerbate s/s)

The laboratory has just called with the arterial blood gas (ABG) results on four patients. Which result is most important for the nurse to report immediately to the health care provider? a. pH 7.34, PaO2 82 mm Hg, PaCO2 40 mm Hg, and O2 sat 97% b. pH 7.35, PaO2 85 mm Hg, PaCO2 45 mm Hg, and O2 sat 95% c. pH 7.46, PaO2 90 mm Hg, PaCO2 32 mm Hg, and O2 sat 98% d. pH 7.31, PaO2 91 mm Hg, PaCO2 50 mm Hg, and O2 sat 96%

d. pH 7.31, PaO2 91 mm Hg, PaCO2 50 mm Hg, and O2 sat 96% (uncompensated respiratory acidosis)

The nurse prepares a patient for synchronized cardioversion knowing that cardioversion differs from defibrillation in that a. defibrillation requires a lower dose of electrical energy. b. cardioversion is indicated to treat atrial bradydysrhythmias. c. defibrillation is synchronized to deliver a shock during the QRS complex. d. patients should be sedated if cardioversion is done on a nonemergency basis.

d. patients should be sedated if cardioversion is done on a nonemergency basis.

After the nurse has finished teaching a patient about the use of sublingual nitroglycerin (Nitrostat), which patient statement indicates that the teaching has been effective? a. "I can expect some nausea as a side effect of nitroglycerin." b. "I should only take the nitroglycerin if I start to have chest pain." c. "I will call an ambulance if I still have pain after taking 3 nitroglycerin 5 minutes apart." d. "Nitroglycerin helps prevent a clot from forming and blocking blood flow to my heart."

"I will call an ambulance if I still have pain after taking 3 nitroglycerin 5 minutes apart."

Triage tags: color meanings

-black: not-salvageable -red: hyper acute (UNCONSCIOUS, tx respiratory, severe blood loss/shock) -yellow: serious (CONSCIOUS head injuries, mod blood loss, spinal cord injury w/ breathing & burns involving respiratory) -green: walking wounded (fx, burns, minor lacerations)

4 ways to prevent pneumonia

1) vaccines 2) turn every 4hrs 3) sit HOB up 4) assessment of immunocompromised

5 rights of delegation

1. Right task 2. Right circumstance (do NOT req independent RN judgment) 3. Right person (qualified & competent) 4. Right direction/communication (clear explanation & about when to report back) 5. Right supervision/evaluation (feedback & pt's outcome)

A patient who is experiencing an acute asthma attack is admitted to the emergency department. Which assessment should the nurse complete first? a. Listen to the patients breath sounds. b. Ask about inhaled corticosteroid use. c. Determine when the dyspnea started. d. Obtain the forced expiratory volume (FEV) flow rate.

a. Listen to the patients breath sounds.

Propranolol (Inderal) is prescribed for a patient diagnosed with hypertension. The nurse should consult with the health care provider before giving this medication when the patient reveals a history of a. asthma. b. daily alcohol use. c. peptic ulcer disease. d. myocardial infarction (MI).

a. asthma

A patient has just been admitted with probable bacterial pneumonia and sepsis. Which order should the nurse implement first? a. Chest x-ray via stretcher b. Blood cultures from two sites c. Ciprofloxacin (Cipro) 400 mg IV d. Acetaminophen (Tylenol) rectal suppository

b. Blood cultures from two sites

Which finding would the nurse expect when assessing the legs of a patient who has a lower motor neuron lesion? a. Spasticity b. Flaccidity c. No sensation d. Hyperactive reflexes

b. Flaccidity

Which action will the nurse plan to take for a 40-year-old patient with multiple sclerosis (MS) who has urinary retention caused by a flaccid bladder? a. Decrease the patients evening fluid intake. b. Teach the patient how to use the Cred method. c. Suggest the use of adult incontinence briefs for nighttime only. d. Assist the patient to the commode every 2 hours during the day.

b. Teach the patient how to use the Cred method.

When the nurse is developing a rehabilitation plan for a 30-year-old patient with a C6 spinal cord injury, an appropriate goal is that the patient will be able to a. drive a car with powered hand controls. b. push a manual wheelchair on a flat surface. c. turn and reposition independently when in bed. d. transfer independently to and from a wheelchair.

b. push a manual wheelchair on a flat surface. (The patient with a C6 injury will be able to use the hands to push a wheelchair on flat, smooth surfaces.)

Which assessment is most important for the nurse to make regarding a patient with myasthenia gravis? a. Pupil size b. Grip strength c. Respiratory effort d. Level of consciousness

c. Respiratory effort

A 68-year-old female patient admitted to the hospital with dehydration is confused and incontinent of urine. Which nursing action will be best to include in the plan of care? a. Restrict fluids between meals and after the evening meal. b. Apply absorbent incontinent pads liberally over the bed linens. c. Insert an indwelling catheter until the symptoms have resolved. d. Assist the patient to the bathroom every 2 hours during the day.

d. Assist the patient to the bathroom every 2 hours during the day.

Tidaling in water seal chamber of chest tube is normal: true or false

true

The nurse obtains a rhythm strip on a patient who has had a myocardial infarction and makes the following analysis: no visible P waves, P-R interval not measurable, ventricular rate 162, R-R interval regular, and QRS complex wide and distorted, QRS duration 0.18 second. The nurse interprets the patients cardiac rhythm as...

v tach

A patient reports feeling numbness and tingling of the left arm before experiencing a tonic-clonic seizure. The nurse determines that this history is consistent with what type of seizure? a. Focal b. Atonic c. Absence d. Myoclonic

a. Focal

A 68-year-old male patient who has bladder cancer had a cystectomy with creation of an Indiana pouch. Which topic will be included in patient teaching? a. Application of ostomy appliances b. Barrier products for skin protection c. Catheterization technique and schedule d. Analgesic use before emptying the pouch

c. Catheterization technique and schedule

Which nursing diagnosis is of highest priority for a patient with Parkinsons disease who is unable to move the facial muscles? a. Activity intolerance b. Self-care deficit: toileting c. Ineffective self-health management d. Imbalanced nutrition: less than body requirements

d. Imbalanced nutrition: less than body requirements

When monitoring for the effectiveness of treatment for a patient with a large anterior wall myocardial infarction, the most important information for the nurse to obtain is a. central venous pressure (CVP). b. systemic vascular resistance (SVR). c. pulmonary vascular resistance (PVR). d. pulmonary artery wedge pressure (PAWP).

d. pulmonary artery wedge pressure (PAWP).

>5mm TB test is positive for what pt?

diabetes & end stage renal failure

To assess whether there is any improvement in a patients dysuria, which question will the nurse ask?

do you have pain when you urinate?

smoking's effect on lungs

inc in cells that create mucus = more mucus than can be excreted

What causes ascites & periorbital edema in nephrotic syndrome?

proteinuria & hypoalbuminuria

nephrotic syndrome s/s

-frothy urine -proteinuria -hypoalbuminuria -hyperCa

COPD typical AGB

-respiratory acidosis -dec O2 & spO2 -inc CO2 (CO2 retention)

The nurse is administering medications to a patient. Which actions by the nurse during this process are consistent with promoting safe delivery of care (select all that apply)? a. Throws away a medication that is not labeled b. Uses a hand sanitizer before preparing a medication c. Identifies the patient by the room number on the door d. Checks lab test results before administering a diuretic e. Gives the patient a list of current medications upon discharge

A, B, D, E -throw away a med that is not labeled -uses a hand sanitizer before preparing a med -check lab results before admin of a diuretic -gives the pt a list of current meds upon DC

When preparing to defibrillate a patient. In which order will the nurse perform the following steps? (Put a comma and a space between each answer choice [A, B, C, D, E].) a. Turn the defibrillator on. b. Deliver the electrical charge. c. Select the appropriate energy level. d. Place the paddles on the patients chest. e. Check the location of other staff and call out all clear.

A, C, D, E, B -Turn the defibrillator on -select appropriate energy level -place paddles on pt's chest -check location of staff & call out clear -deliver eléctrical charge

The nurse has reinforced home care instructions to a client who had a permanent pacemaker inserted. Which educational outcome has the greatest impact on the client's long-term cardiac health? A. Knowledge of when it is safe to resume sexual activity B. The ability to take an accurate pulse in either the wrist or neck C. An understanding of the importance of proper microwave oven usage D. An understanding of why vigorous arm and shoulder movement must be avoided initially

B. The ability to take an accurate pulse in either the wrist or neck

The following four patients arrive in the emergency department (ED) after a motor vehicle collision. In which order should the nurse assess them? a. A 74-year-old with palpitations and chest pain b. A 43-year-old complaining of 7/10 abdominal pain c. A 21-year-old with multiple fractures of the face and jaw d. A 37-year-old with a misaligned left leg with intact pulses

C, A, B, D -a 21yo w/ mult fx of face & jaw (airway obstruction) -74yo w/ palpitations & chest pain (cardiac ischemia) -43yo c/o 7/10 ab pain (ab trauma/bleed) -37yo w/ misaligned L leg w/ intact pulses (poss fx)

A 27-year-old patient who has been treated for status epilepticus in the emergency department will be transferred to the medical nursing unit. Which equipment should the nurse have available in the patients assigned room (select all that apply)? a. Side-rail pads b. Tongue blade c. Oxygen mask d. Suction tubing e. Urinary catheter f. Nasogastric tube

a, c, d -side-rail pads -oxygen mask -suction tubing

The nurse in the emergency department receives arterial blood gas results for four recently admitted patients with obstructive pulmonary disease. Which patient will require the most rapid action by the nurse? a. 22-year-old with ABG results: pH 7.28, PaCO2 60 mm Hg, and PaO2 58 mm Hg b. 34-year-old with ABG results: pH 7.48, PaCO2 30 mm Hg, and PaO2 65 mm Hg c. 45-year-old with ABG results: pH 7.34, PaCO2 33 mm Hg, and PaO2 80 mm Hg d. 65-year-old with ABG results: pH 7.31, PaCO2 58 mm Hg, and PaO2 64 mm Hg

a. 22-year-old with ABG results: pH 7.28, PaCO2 60 mm Hg, and PaO2 58 mm Hg (severe uncompensated respiratory acidosis & hypoxemia)

After the return of spontaneous circulation following the resuscitation of a patient who had a cardiac arrest, therapeutic hypothermia is ordered. Which action will the nurse include in the plan of care? a. Apply external cooling device. b. Check mental status every 15 minutes. c. Avoid the use of sedative medications. d. Rewarm if temperature is <91 F (32.8 C).

a. Apply external cooling device.

Which action will the nurse include in the plan of care for a 62-year-old patient who is experiencing pain from trigeminal neuralgia? a. Assess fluid and dietary intake. b. Apply ice packs for 20 minutes. c. Teach facial relaxation techniques. d. Spend time talking with the patient.

a. Assess fluid and dietary intake. (The patient with an acute episode of trigeminal neuralgia may be unwilling to eat or drink, so assessment of nutritional and hydration status is important. Because stimulation by touch is the precipitating factor for pain, relaxation of the facial muscles will not improve symptoms. Application of ice is likely to precipitate pain. The patient will not want to engage in conversation, which may precipitate attacks.)

Two days after an acute myocardial infarction (MI), a patient complains of stabbing chest pain that increases with a deep breath. Which action will the nurse take first? a. Auscultate the heart sounds. b. Check the patients temperature. c. Notify the patients health care provider. d. Give the PRN acetaminophen (Tylenol).

a. Auscultate the heart sounds.

Following a cauda equina spinal cord injury, which action will the nurse include in the plan of care? a. Catheterize patient every 3 to 4 hours. b. Assist patient to ambulate several times daily. c. Administer medications to reduce bladder spasm. d. Stabilize the neck when repositioning the patient.

a. Catheterize patient every 3 to 4 hours.

When assessing an older patient admitted to the emergency department (ED) with a broken arm and facial bruises, the nurse observes several additional bruises in various stages of healing. Which statement or question by the nurse is most appropriate? a. Do you feel safe in your home? b. You should not return to your home. c. Would you like to see a social worker? d. I need to report my concerns to the police.

a. Do you feel safe in your home?

A patient is admitted with possible botulism poisoning after eating home-canned green beans. Which intervention ordered by health care provider will the nurse question? a. Encourage oral fluids to 3 L/day b. Document neurologic symptoms c. Position patient lying on the side d. Observe respiratory status closely

a. Encourage oral fluids to 3 L/day (The patient should be maintained on NPO status because neuromuscular weakness increases risk for aspiration.)

A 20-year-old patient arrives in the emergency department (ED) several hours after taking 25 to 30 acetaminophen (Tylenol) tablets. Which action will the nurse plan to take? a. Give N-acetylcysteine (Mucomyst). b. Discuss the use of chelation therapy. c. Start oxygen using a non-rebreather mask. d. Have the patient drink large amounts of water.

a. Give N-acetylcysteine (Mucomyst).

Which statement by a patient scheduled for surgery is most important to report to the health care provider? a. I had a heart valve replacement last year. b. I had bacterial pneumonia 3 months ago. c. I have knee pain whenever I walk or jog. d. I have a strong family history of breast cancer.

a. I had a heart valve replacement last year. (at risk for endocarditis r/t sx may need prophylactic ATBx)

Which action by a new registered nurse (RN) who is orienting to the progressive care unit indicates a good understanding of the treatment of cardiac dysrhythmias? a. Injects IV adenosine (Adenocard) over 2 seconds to a patient with supraventricular tachycardia b. Obtains the defibrillator and quickly brings it to the bedside of a patient whose monitor shows asystole c. Turns the synchronizer switch to the on position before defibrillating a patient with ventricular fibrillation d. Gives the prescribed dose of diltiazem (Cardizem) to a patient with new-onset type II second degree AV block

a. Injects IV adenosine (Adenocard) over 2 seconds to a patient with supraventricular tachycardia

A 31-year-old woman who has multiple sclerosis (MS) asks the nurse about risks associated with pregnancy. Which response by the nurse is accurate? a. MS symptoms may be worse after the pregnancy. b. Women with MS frequently have premature labor. c. MS is associated with an increased risk for congenital defects. d. Symptoms of MS are likely to become worse during pregnancy.

a. MS symptoms may be worse after the pregnancy.

Which nursing responsibilities are priorities when caring for patient returning from a cardiac catherization (select all that apply)? a. Monitoring vital signs and ECG b. Checking the catheter insertion site and distal pulses c. assisting the patient to ambulate to the bathroom to void d. informing the patient that he will be sleepy from the general anesthesia e. instructing the patient about the risks of the radioactive isotope injection.

a. Monitoring vital signs and ECG b. Checking the catheter insertion site and distal pulses

The nurse cares for a patient who has just had a thoracentesis. Which assessment information obtained by the nurse is a priority to communicate to the health care provider? a. Oxygen saturation is 88%. b. Blood pressure is 145/90 mm Hg. c. Respiratory rate is 22 breaths/minute when lying flat. d. Pain level is 5 (on 0 to 10 scale) with a deep breath.

a. Oxygen saturation is 88%. (pneumothorax might be occurring, spO2 should inc)

To determine whether there is a delay in impulse conduction through the atria, the nurse will measure the duration of the patients a. P wave. b. Q wave. c. P-R interval. d. QRS complex.

a. P wave

A nurse who works on the neurology unit just received change-of-shift report. Which patient will the nurse assess first? a. Patient with botulism who is experiencing difficulty swallowing b. Patient with Bells palsy who has herpes vesicles in front of the ear c. Patient with neurosyphilis who has tabes dorsalis and decreased deep tendon reflexes d. Patient with an abscess caused by injectable drug use who needs tetanus immune globulin

a. Patient with botulism who is experiencing difficulty swallowing

After change-of-shift report, which patient should the nurse assess first? a. Patient with myasthenia gravis who is reporting increased muscle weakness b. Patient with a bilateral headache described as like a band around my head c. Patient with seizures who is scheduled to receive a dose of phenytoin (Dilantin) d. Patient with Parkinsons disease who has developed cogwheel rigidity of the arms

a. Patient with myasthenia gravis who is reporting increased muscle weakness (Because increased muscle weakness may indicate the onset of a myasthenic crisis, the nurse should assess this patient first.)

The following interventions are part of the emergency department (ED) protocol for a patient who has been admitted with multiple bee stings to the hands. Which action should the nurse take first? a. Remove the patients rings. b. Apply ice packs to both hands. c. Apply calamine lotion to any itching areas. d. Give diphenhydramine (Benadryl) 50 mg PO.

a. Remove the patients rings.

Which integumentary assessment data from an older patient admitted with bacterial pneumonia is of most concern for the nurse? a. Reports a history of allergic rashes b. Scattered macular brown areas on extremities c. Skin brown and wrinkled, skin tenting on forearm d. Longitudinal nail bed ridges noted; sparse scalp hair

a. Reports a history of allergic rashes

The nurse assesses a patient with chronic obstructive pulmonary disease (COPD) who has been admitted with increasing dyspnea over the last 3 days. Which finding is most important for the nurse to report to the health care provider? a. Respirations are 36 breaths/minute. b. Anterior-posterior chest ratio is 1:1. c. Lung expansion is decreased bilaterally. d. Hyperresonance to percussion is present.

a. Respirations are 36 breaths/minute

After reviewing the health record shown in the accompanying figure for a patient who has multiple risk factors for Alzheimers disease, which topic will be most important for the nurse to discuss with the patient? a. Tobacco use b. Family history c. Head injury history d. Total cholesterol level

a. Tobacco use

Which information should the nurse include when teaching a patient who has just received a prescription for ciprofloxacin (Cipro) to treat a urinary tract infection? a. Use a sunscreen with a high SPF when exposed to the sun. b. Sun exposure may decrease the effectiveness of the medication. c. Photosensitivity may result in an artificial-looking tan appearance. d. Wear sunglasses to avoid eye damage while taking this medication.

a. Use a sunscreen with a high SPF when exposed to the sun.

Following assessment of a patient with pneumonia, the nurse identifies a nursing diagnosis of ineffective airway clearance. Which assessment data best supports this diagnosis? a. Weak, nonproductive cough effort b. Large amounts of greenish sputum c. Respiratory rate of 28 breaths/minute d. Resting pulse oximetry (SpO2) of 85%

a. Weak, nonproductive cough effort

A young adult patient who denies any history of smoking is seen in the clinic with a new diagnosis of chronic obstructive pulmonary disease (COPD). It is most appropriate for the nurse to teach the patient about a. a1-antitrypsin testing. b. use of the nicotine patch. c. continuous pulse oximetry. d. effects of leukotriene modifiers.

a. a1-antitrypsin testing.

A patient with dilated cardiomyopathy has new onset atrial fibrillation that has been unresponsive to drug therapy for several days. The priority teaching needed for this patient would include information about a. anticoagulant therapy. b. permanent pacemakers. c. electrical cardioversion. d. IV adenosine (Adenocard).

a. anticoagulant therapy. (Afib for more than 48hrs req 3wks of anticoagulant tx before attempting cardio version)

An unresponsive 79-year-old is admitted to the emergency department (ED) during a summer heat wave. The patients core temperature is 105.4 F (40.8 C), blood pressure (BP) 88/50, and pulse 112. The nurse initiallywill plan to a. apply wet sheets and a fan to the patient. b. provide O2 at 6 L/min with a nasal cannula. c. start lactated Ringers solution at 1000 mL/hr. d. give acetaminophen (Tylenol) rectal suppository.

a. apply wet sheets and a fan to the patient.

A triage nurse in a busy emergency department (ED) assesses a patient who complains of 7/10 abdominal pain and states, I had a temperature of 103.9 F (39.9 C) at home. The nurses first action should be to a. assess the patients current vital signs. b. give acetaminophen (Tylenol) per agency protocol. c. ask the patient to provide a clean-catch urine for urinalysis. d. tell the patient that it will 1 to 2 hours before being seen by the doctor.

a. assess the patients current vital signs

A triage nurse in a busy emergency department (ED) assesses a patient who complains of 7/10 abdominal pain and states, I had a temperature of 103.9 F (39.9 C) at home. The nurses first action should be to a. assess the patients current vital signs. b. give acetaminophen (Tylenol) per agency protocol. c. ask the patient to provide a clean-catch urine for urinalysis. d. tell the patient that it will 1 to 2 hours before being seen by the doctor.

a. assess the patients current vital signs.

A patient who is lethargic and exhibits deep, rapid respirations has the following arterial blood gas (ABG) results: pH 7.32, PaO2 88 mm Hg, PaCO2 37 mm Hg, and HCO3 16 mEq/L. How should the nurse interpret these results? a. Metabolic acidosis b. Metabolic alkalosis c. Respiratory acidosis d. Respiratory alkalosis

a. metabolic acidosis

A 32-year-old pregnant patient with Bells palsy refuses to eat while others are present because of embarrassment about drooling. The best response by the nurse is to a. respect the patients feelings and arrange for privacy at mealtimes. b. teach the patient to chew food on the unaffected side of the mouth. c. offer the patient liquid nutritional supplements at frequent intervals. d. discuss the patients concerns with visitors who arrive at mealtimes.

a. respect the patients feelings and arrange for privacy at mealtimes.

To assess the functioning of the trigeminal and facial nerves (CNs V and VII), the nurse should a. shine a light into the patients pupil. b. check for unilateral eyelid drooping. c. touch a cotton wisp strand to the cornea. d. have the patient read a magazine or book.

a. shine a light into the patients pupil.

When planning the response to the potential use of smallpox as an agent of terrorism, the emergency department (ED) nurse manager will plan to obtain adequate quantities of a. vaccine. b. atropine. c. antibiotics. d. whole blood.

a. vaccine

The nurse is caring for a patient who was admitted to the coronary care unit following an acute myocardial infarction (AMI) and percutaneous coronary intervention the previous day. Teaching for this patient would include a. when cardiac rehabilitation will begin. b. the typical emotional responses to AMI. c. information regarding discharge medications. d. the pathophysiology of coronary artery disease.

a. when cardiac rehab will begin

Which patient teaching points should the nurse include when providing discharge instructions to a patient with a new permanent pacemaker and the caregiver (select all that apply)? a. Avoid or limit air travel. b. Take and record a daily pulse rate. c. Obtain and wear a Medic Alert ID device at all times. d. Avoid lifting arm on the side of the pacemaker above shoulder. e. Avoid microwave ovens because they interfere with pacemaker function.

b, c, d -take & record daily pulse -obtain & wear medical alert bracelet -avoid lifting arm on side of PM above shoulder

A patient is being discharged after the insertion of a permanent pacemaker. Which statement made by the patient indicates an understanding regarding appropriate self-care? a. "Every morning I will perform arm and shoulder stretches." b. "Each day I'll take my pulse and record it in a log." c. "I'll have to get rid of my microwave oven." d. "I won't be able to use my electric blanket anymore."

b. "Each day I'll take my pulse and record it in a log."

After the nurse teaches the patient about the use of carvedilol (Coreg) in preventing anginal episodes, which statement by a patient indicates that the teaching has been effective? a. "Carvedilol will help my heart muscle work harder." b. "It is important not to suddenly stop taking the carvedilol." c. "I can expect to feel short of breath when taking carvedilol." d. "Carvedilol will increase the blood flow to my heart muscle."

b. "It is important not to suddenly stop taking the carvedilol."

The nurse monitors a patient after chest tube placement for a hemopneumothorax. The nurse is most concerned if which assessment finding is observed? a. A large air leak in the water-seal chamber b. 400 mL of blood in the collection chamber c. Complaint of pain with each deep inspiration d. Subcutaneous emphysema at the insertion site

b. 400 mL of blood in the collection chamber

After 2 months of tuberculosis (TB) treatment with isoniazid (INH), rifampin (Rifadin), pyrazinamide (PZA), and ethambutol, a patient continues to have positive sputum smears for acid-fast bacilli (AFB). Which action should the nurse take next? a. Teach about treatment for drug-resistant TB treatment. b. Ask the patient whether medications have been taken as directed. c. Schedule the patient for directly observed therapy three times weekly. d. Discuss with the health care provider the need for the patient to use an injectable antibiotic.

b. Ask the patient whether medications have been taken as directed. (pt's struggle w/ drug compliance d/t the large volume of drugs)

Several weeks after a stroke, a 50-year-old male patient has impaired awareness of bladder fullness, resulting in urinary incontinence. Which nursing intervention will be best to include in the initial plan for an effective bladder training program? a. Limit fluid intake to 1200 mL daily to reduce urine volume. b. Assist the patient onto the bedside commode every 2 hours. c. Perform intermittent catheterization after each voiding to check for residual urine. d. Use an external condom catheter to protect the skin and prevent embarrassment.

b. Assist the patient onto the bedside commode every 2 hours.

The oxygen saturation (SpO2) for a patient with left lower lobe pneumonia is 90%. The patient has rhonchi, a weak cough effort, and complains of fatigue. Which action is a priority for the nurse to take? a. Position the patient on the left side. b. Assist the patient with staged coughing. c. Place a humidifier in the patients room. d. Schedule a 2-hour rest period for the patient.

b. Assist the patient with staged coughing.

Gastric lavage and administration of activated charcoal are ordered for an unconscious patient who has been admitted to the emergency department (ED) after ingesting 30 lorazepam (Ativan) tablets. Which action should the nurse plan to do first? a. Insert a large-bore orogastric tube. b. Assist with intubation of the patient. c. Prepare a 60-mL syringe with saline. d. Give first dose of activated charcoal.

b. Assist with intubation of the patient.

19. A patient is admitted to the emergency department (ED) after falling through the ice while ice skating. Which assessment will the nurse obtain first? a. Heart rate b. Breath sounds c. Body temperature d. Level of consciousness

b. Breath sounds

A patients complete blood count (CBC) shows a hemoglobin of 19 g/dL and a hematocrit of 54%. Which question should the nurse ask to determine possible causes of this finding? a. Have you had a recent weight loss? b. Do you have any history of lung disease? c. Have you noticed any dark or bloody stools? d. What is your dietary intake of meats and protein?

b. Do you have any history of lung disease? (polycythemia indicative of COPD)

A 63-year-old patient with primary hyperparathyroidism has a serum phosphorus level of 1.7 mg/dL (0.55) mmol/L) and calcium of 14 mg/dL (3.5 mmol/L). Which nursing action should be included in the plan of care? a. Restrict the patient to bed rest. b. Encourage 4000 mL of fluids daily. c. Institute routine seizure precautions. d. Assess for positive Chvosteks sign.

b. Encourage 4000 mL of fluids daily. (at risk for kidney stones)

A 58-year-old male patient who is diagnosed with nephrotic syndrome has ascites and 4+ leg edema. Which nursing diagnosis is a priority for the patient? a. Activity intolerance related to rapidly increased weight b. Excess fluid volume related to low serum protein levels c. Disturbed body image related to peripheral edema and ascites d. Altered nutrition: less than required related to protein restriction

b. Excess fluid volume related to low serum protein levels

A 24-year-old woman with Crohns disease develops a fever and symptoms of a urinary tract infection (UTI) with tan, fecal-smelling urine. What information will the nurse add to a general teaching plan about UTIs in order to individualize the teaching for this patient? a. Bacteria in the perianal area can enter the urethra. b. Fistulas can form between the bowel and bladder. c. Drink adequate fluids to maintain normal hydration. d. Empty the bladder before and after sexual intercourse.

b. Fistulas can form between the bowel and bladder.

A home care nurse is planning care for a patient who has just been diagnosed with type 2 diabetes mellitus. Which task is appropriate for the nurse to delegate to the home health aide? a. Assist the patient to choose appropriate foods. b. Help the patient with a daily bath and oral care. c. Check the patients feet for signs of breakdown. d. Teach the patient how to monitor blood glucose.

b. Help the patient with a daily bath and oral care.

The nurse provides dietary teaching for a patient with chronic obstructive pulmonary disease (COPD) who has a low body mass index (BMI). Which patient statement indicates that the teaching has been effective? a. I will drink lots of fluids with my meals. b. I can have ice cream as a snack every day. c. I will exercise for 15 minutes before meals. d. I will decrease my intake of meat and poultry.

b. I can have ice cream as a snack every day (High-calorie foods like ice cream are an appropriate snack for patients with COPD. Fluid intake of 3 L/day is recommended, but fluids should be taken between meals rather than with meals to improve oral intake of solid foods. The patient should avoid exercise for an hour before meals to prevent fatigue while eating. Meat and dairy products are high in protein and are good choices for the patient with COPD.)

The nurse provides discharge instructions to a patient who was hospitalized for pneumonia. Which statement, if made by the patient, indicates a good understanding of the instructions? a. I will call the doctor if I still feel tired after a week. b. I will continue to do the deep breathing and coughing exercises at home. c. I will schedule two appointments for the pneumonia and influenza vaccines. d. Ill cancel my chest x-ray appointment if Im feeling better in a couple weeks.

b. I will continue to do the deep breathing and coughing exercises at home.

The clinic nurse makes a follow-up telephone call to a patient with asthma. The patient reports having a baseline peak flow reading of 600 L/minute and the current peak flow is 420 L/minute. Which action should the nurse takefirst? a. Tell the patient to go to the hospital emergency department. b. Instruct the patient to use the prescribed albuterol (Proventil). c. Ask about recent exposure to any new allergens or asthma triggers. d. Question the patient about use of the prescribed inhaled corticosteroids.

b. Instruct the patient to use the prescribed albuterol (Proventil). (70% of normal indicates need for SABAs)

A patient who has severe Alzheimers disease (AD) is being admitted to the hospital for surgery. Which intervention will the nurse include in the plan of care? a. Encourage the patient to discuss events from the past. b. Maintain a consistent daily routine for the patients care. c. Reorient the patient to the date and time every 2 to 3 hours. d. Provide the patient with current newspapers and magazines.

b. Maintain a consistent daily routine for the patients care.

Following a thymectomy, a 62-year-old male patient with myasthenia gravis receives the usual dose of pyridostigmine (Mestinon). An hour later, the patient complains of nausea and severe abdominal cramps. Which action should the nurse take first? a. Auscultate the patients bowel sounds. b. Notify the patients health care provider. c. Administer the prescribed PRN antiemetic drug. d. Give the scheduled dose of prednisone (Deltasone).

b. Notify the patients health care provider. (s/s indicate possible cholinergic crisis, likely atropine will be Rx)

During the primary assessment of a victim of a motor vehicle collision, the nurse determines that the patient is breathing and has an unobstructed airway. Which action should the nurse take next? a. Palpate extremities for bilateral pulses. b. Observe the patients respiratory effort. c. Check the patients level of consciousness. d. Examine the patient for any external bleeding.

b. Observe the patients respiratory effort.

The emergency department nurse is evaluating the effectiveness of therapy for a patient who has received treatment during an asthma attack. Which assessment finding is the best indicator that the therapy has been effective? a. No wheezes are audible. b. Oxygen saturation is >90%. c. Accessory muscle use has decreased. d. Respiratory rate is 16 breaths/minute.

b. Oxygen saturation is >90%.

A patient is admitted to the emergency department with an open stab wound to the left chest. What is the first action that the nurse should take? a. Position the patient so that the left chest is dependent. b. Tape a nonporous dressing on three sides over the chest wound. c. Cover the sucking chest wound firmly with an occlusive dressing. d. Keep the head of the patients bed at no more than 30 degrees elevation.

b. Tape a nonporous dressing on three sides over the chest wound.

A patient with chronic obstructive pulmonary disease (COPD) has rhonchi throughout the lung fields and a chronic, nonproductive cough. Which nursing intervention will be most effective? a. Change the oxygen flow rate to the highest prescribed rate. b. Teach the patient to use the Flutter airway clearance device. c. Reinforce the ongoing use of pursed lip breathing techniques. d. Teach the patient about consistent use of inhaled corticosteroids.

b. Teach the patient to use the Flutter airway clearance device. (Airway clearance devices assist with moving mucus into larger airways where it can more easily be expectorated.)

Which assessment data collected by the nurse who is admitting a patient with chest pain suggest that the pain is caused by an acute myocardial infarction (AMI)? a. The pain increases with deep breathing. b. The pain has lasted longer than 30 minutes. c. The pain is relieved after the patient takes nitroglycerin. d. The pain is reproducible when the patient raises the arms.

b. The pain has lasted longer than 30 minutes.

The health care provider is considering the use of sumatriptan (Imitrex) for a 54-year-old male patient with migraine headaches. Which information obtained by the nurse is most important to report to the health care provider? a. The patient drinks 1 to 2 cups of coffee daily. b. The patient had a recent acute myocardial infarction. c. The patient has had migraine headaches for 30 years. d. The patient has taken topiramate (Topamax) for 2 months.

b. The patient had a recent acute myocardial infarction. (vasoconstrictors should be avoided in those w/ CAD)

Which assessment data for a patient who has Guillain-Barr syndrome will require the nurses most immediate action? a. The patients triceps reflexes are absent. b. The patient is continuously drooling saliva. c. The patient complains of severe pain in the feet. d. The patients blood pressure (BP) is 150/82 mm Hg.

b. The patient is continuously drooling saliva.

A patient is scheduled for a cardiac catheterization with coronary angiography. Before the test, the nurse informs the patient that a. it will be important to lie completely still during the procedure. b. a flushed feeling may be noted when the contrast dye is injected. c. monitored anesthesia care will be provided during the procedure. d. arterial pressure monitoring will be required for 24 hours after the test.

b. a flushed feeling may be noted when the contrast dye is injected.

A patient who has experienced blunt abdominal trauma during a motor vehicle collision is complaining of increasing abdominal pain. The nurse will plan to teach the patient about the purpose of a. peritoneal lavage. b. abdominal ultrasonography. c. nasogastric (NG) tube placement. d. magnetic resonance imaging (MRI).

b. abdominal ultrasonography.

When a 74-year-old patient is seen in the health clinic with new development of a stooped posture, shuffling gait, and pill rollingtype tremor, the nurse will anticipate teaching the patient about a. oral corticosteroids. b. antiparkinsonian drugs. c. magnetic resonance imaging (MRI). d. electroencephalogram (EEG) testing.

b. antiparkinsonian drugs (The diagnosis of Parkinsons is made when two of the three characteristic manifestations of tremor, rigidity, and bradykinesia are present. The confirmation of the diagnosis is made on the basis of improvement when antiparkinsonian drugs are administered.)

The nurse will plan to teach a 27-year-old female who smokes 2 packs of cigarettes daily about the increased risk for a. kidney stones. b. bladder cancer. c. bladder infection. d. interstitial cystitis.

b. bladder CA

The nurse will suspect that the patient with stable angina is experiencing a side effect of the prescribed metoprolol (Lopressor) if the a. patient is restless and agitated. b. blood pressure is 90/54 mm Hg. c. patient complains about feeling anxious. d. cardiac monitor shows a heart rate of 61 beats/minute.

b. blood pressure is 90/54 mm Hg.

Diltiazem (Cardizem) is ordered for a patient with newly diagnosed Prinzmetal's (variant) angina. When teaching the patient, the nurse will include the information that diltiazem will a. reduce heart palpitations. b. decrease spasm of the coronary arteries. c. increase the force of the heart contractions. d. help prevent plaque from forming in the coronary arteries.

b. decrease spasm of the coronary arteries.

When obtaining a health history and physical assessment for a 36-year-old female patient with possible multiple sclerosis (MS), the nurse should a. assess for the presence of chest pain. b. inquire about urinary tract problems. c. inspect the skin for rashes or discoloration. d. ask the patient about any increase in libido.

b. inquire about urinary tract problems. (Urinary tract problems with incontinence or retention are common symptoms of MS.)

A diabetic patients arterial blood gas (ABG) results are pH 7.28; PaCO2 34 mm Hg; PaO2 85 mm Hg; HCO3 18 mEq/L. The nurse would expect which finding? a. Intercostal retractions b. Kussmaul respirations c. Low oxygen saturation (SpO2) d. Decreased venous O2 pressure

b. kussmaul respirations

The nurse in the outpatient clinic has obtained health histories for these new patients. Which patient may need referral for genetic testing? a. 35-year-old patient whose maternal grandparents died after strokes at ages 90 and 96 b. 18-year-old patient with a positive pregnancy test whose first child has cerebral palsy c. 34-year-old patient who has a sibling with newly diagnosed polycystic kidney disease d. 50-year-old patient with a history of cigarette smoking who is complaining of dyspnea

c. 34-year-old patient who has a sibling with newly diagnosed polycystic kidney disease (The adult form of polycystic kidney disease is an autosomal dominant disorder and frequently it is asymptomatic until the patient is older. Presymptomatic testing will give the patient information that will be useful in guiding lifestyle and childbearing choices.)

A 19-year-old patient with massive trauma and possible spinal cord injury is admitted to the emergency department (ED). Which assessment finding by the nurse will help confirm a diagnosis of neurogenic shock? a. Inspiratory crackles. b. Cool, clammy extremities. c. Apical heart rate 45 beats/min. d. Temperature 101.2 F (38.4 C).

c. Apical heart rate 45 beats/min.

Family members are in the patients room when the patient has a cardiac arrest and the staff start resuscitation measures. Which action should the nurse take next? a. Keep the family in the room and assign a staff member to explain the care given and answer questions. b. Ask the family to wait outside the patients room with a designated staff member to provide emotional support. c. Ask the family members about whether they would prefer to remain in the patients room or wait outside the room. d. Tell the family members that patients are comforted by having family members present during resuscitation efforts.

c. Ask the family members about whether they would prefer to remain in the patients room or wait outside the room.

A 22-year-old patient seen at the health clinic with a severe migraine headache tells the nurse about having other similar headaches recently. Which initial action should the nurse take? a. Teach about the use of triptan drugs. b. Refer the patient for stress counseling. c. Ask the patient to keep a headache diary. d. Suggest the use of muscle-relaxation techniques.

c. Ask the patient to keep a headache diary.

When administering the Mini-Cog exam to a patient with possible Alzheimers disease, which action will the nurse take? a. Check the patients orientation to time and date. b. Obtain a list of the patients prescribed medications. c. Ask the person to use a clock drawing to indicate a specific time. d. Determine the patients ability to recognize a common object such as a pen.

c. Ask the person to use a clock drawing to indicate a specific time.

Which nursing action will the home health nurse include in the plan of care for a patient with paraplegia at the T4 level in order to prevent autonomic dysreflexia? a. Support selection of a high-protein diet. b. Discuss options for sexuality and fertility. c. Assist in planning a prescribed bowel program. d. Use quad coughing to strengthen cough efforts.

c. Assist in planning a prescribed bowel program. (Fecal impaction is a common stimulus for autonomic dysreflexia. Dietary protein, coughing, and discussing sexuality/fertility should be included in the plan of care but will not reduce the risk for autonomic dysreflexia.)

Which information about a patient with Goodpasture syndrome requires the most rapid action by the nurse? a. Blood urea nitrogen level is 70 mg/dL. b. Urine output over the last 2 hours is 30 mL. c. Audible crackles bilaterally over the posterior chest to the midscapular level. d. Elevated level of antiglomerular basement membrane (anti-GBM) antibodies.

c. Audible crackles bilaterally over the posterior chest to the midscapular level.

A 22-year-old patient who experienced a near drowning accident in a local pool, but now is awake and breathing spontaneously, is admitted for observation. Which assessment will be most important for the nurse to take during the observation period? a. Auscultate heart sounds. b. Palpate peripheral pulses. c. Auscultate breath sounds. d. Check pupil reaction to light

c. Auscultate breath sounds.

A patient is recovering from a myocardial infarction (MI) and develops chest pain on day 3 that increases when taking a deep breath and is relieved by leaning forward. Which action should the nurse take next? a. Assess the feet for pedal edema. b. Palpate the radial pulses bilaterally. c. Auscultate for a pericardial friction rub. d. Check the heart monitor for dysrhythmias.

c. Auscultate for a pericardial friction rub. (s/s consistent w/ pericarditis)

Which action should the nurse take first when a patient complains of acute chest pain and dyspnea soon after insertion of a centrally inserted IV catheter? a. Notify the health care provider. b. Offer reassurance to the patient. c. Auscultate the patients breath sounds. d. Give the prescribed PRN morphine sulfate IV.

c. Auscultate the patients breath sounds.

Nursing staff on a hospital unit are reviewing rates of hospital-acquired infections (HAI) of the urinary tract. Which nursing action will be most helpful in decreasing the risk for HAI in patients admitted to the hospital? a. Encouraging adequate oral fluid intake b. Testing urine with a dipstick daily for nitrites c. Avoiding unnecessary urinary catheterizations d. Providing frequent perineal hygiene to patients

c. Avoiding unnecessary urinary catheterizations

What is the primary BP effect of β-adrenergic blockers such as atenolol (Tenormin)? a. Vasodilation of arterioles by blocking movement of calcium into cells b. Decrease Na+ and water reabsorption by blocking the effect of aldosterone c. Decrease CO by decreasing rate and strength of the heart and renin secretion by the kidneys d. Vasodilation caused by inhibiting sympathetic outflow from the central nervous system (CNS)

c. Cardioselective β-adrenergic blockers decrease CO, reduce sympathetic vasoconstrictor tone, and decrease renin secretion by kidneys

A 28-year-old male patient is diagnosed with polycystic kidney disease. Which information is most appropriate for the nurse to include in teaching at this time? a. Complications of renal transplantation b. Methods for treating severe chronic pain c. Discussion of options for genetic counseling d. Differences between hemodialysis and peritoneal dialysis

c. Discussion of options for genetic counseling

A patient with chronic obstructive pulmonary disease (COPD) has poor gas exchange. Which action by the nurse would be most appropriate? a. Have the patient rest in bed with the head elevated to 15 to 20 degrees. b. Ask the patient to rest in bed in a high-Fowlers position with the knees flexed. c. Encourage the patient to sit up at the bedside in a chair and lean slightly forward. d. Place the patient in the Trendelenburg position with several pillows behind the head.

c. Encourage the patient to sit up at the bedside in a chair and lean slightly forward.

The nurse provides preoperative instruction for a patient scheduled for a left pneumonectomy for cancer of the lung. Which information should the nurse include about the patients postoperative care? a. Positioning on the right side b. Bed rest for the first 24 hours c. Frequent use of an incentive spirometer d. Chest tube placement with continuous drainage

c. Frequent use of an incentive spirometer

A 32-year-old patient with a history of polycystic kidney disease is admitted to the surgical unit after having shoulder surgery. Which of the routine postoperative orders is most important for the nurse to discuss with the health care provider? a. Infuse 5% dextrose in normal saline at 75 mL/hr. b. Order regular diet after patient is awake and alert. c. Give ketorolac (Toradol) 10 mg PO PRN for pain. d. Draw blood urea nitrogen (BUN) and creatinine in 2 hours

c. Give ketorolac (Toradol) 10 mg PO PRN for pain.

The nurse is admitting a patient with a neck fracture at the C6 level to the intensive care unit. Which assessment finding(s) indicate(s) neurogenic shock? a. Hyperactive reflex activity below the level of injury b. Involuntary, spastic movements of the arms and legs c. Hypotension, bradycardia, and warm, pink extremities d. Lack of sensation or movement below the level of injury

c. Hypotension, bradycardia, and warm, pink extremities

A patient with hypotension and an elevated temperature after working outside on a hot day is treated in the emergency department (ED). The nurse determines that discharge teaching has been effective when the patient makes which statement? a. I will take salt tablets when I work outdoors in the summer. b. I should take acetaminophen (Tylenol) if I start to feel too warm. c. I should drink sports drinks when working outside in hot weather. d. I will move to a cool environment if I notice that I am feeling confused.

c. I should drink sports drinks when working outside in hot weather.

The nurse determines that further instruction is needed for a patient with interstitial cystitis when the patient says which of the following? a. I should stop having coffee and orange juice for breakfast. b. I will buy calcium glycerophosphate (Prelief) at the pharmacy. c. I will start taking high potency multiple vitamins every morning. d. I should call the doctor about increased bladder pain or odorous urine.

c. I will start taking high potency multiple vitamins every morning. (High-potency multiple vitamins may irritate the bladder and increase symptoms)

In preparation for discharge, the nurse teaches a patient with chronic stable angina how to use the prescribed short-acting and long-acting nitrates. Which patient statement indicates that the teaching has been effective? a. I will check my pulse rate before I take any nitroglycerin tablets. b. I will put the nitroglycerin patch on as soon as I get any chest pain. c. I will stop what I am doing and sit down before I put the nitroglycerin under my tongue. d. I will be sure to remove the nitroglycerin patch before taking any sublingual nitroglycerin.

c. I will stop what I am doing and sit down before I put the nitroglycerin under my tongue.

Which statement by a patient with restrictive cardiomyopathy indicates that the nurses discharge teaching about self-management has been most effective? a. I will avoid taking aspirin or other antiinflammatory drugs. b. I will need to limit my intake of salt and fluids even in hot weather. c. I will take antibiotics when my teeth are cleaned at the dental office. d. I should begin an exercise program that includes things like biking or swimming.

c. I will take antibiotics when my teeth are cleaned at the dental office.

A 46-year-old female patient returns to the clinic with recurrent dysuria after being treated with trimethoprim and sulfamethoxazole (Bactrim) for 3 days. Which action will the nurse plan to take? a. Teach the patient to take the prescribed Bactrim for 3 more days. b. Remind the patient about the need to drink 1000 mL of fluids daily. c. Obtain a midstream urine specimen for culture and sensitivity testing. d. Suggest that the patient use acetaminophen (Tylenol) to treat the symptoms.

c. Obtain a midstream urine specimen for culture and sensitivity testing. (Because uncomplicated urinary tract infections (UTIs) are usually successfully treated with 3 days of antibiotic therapy, this patient will need a urine culture and sensitivity to determine appropriate antibiotic therapy.)

A nurse is caring for a patient with heart failure. Which task is appropriate for the nurse to delegate to experienced unlicensed assistive personnel (UAP)? a. Monitor for shortness of breath or fatigue after ambulation. b. Instruct the patient about the need to alternate activity and rest. c. Obtain the patients blood pressure and pulse rate after ambulation. d. Determine whether the patient is ready to increase the activity level.

c. Obtain the patients blood pressure and pulse rate after ambulation.

A 71-year-old patient with Alzheimers disease (AD) who is being admitted to a long-term care facility has had several episodes of wandering away from home. Which action will the nurse include in the plan of care? a. Reorient the patient several times daily. b. Have the family bring in familiar items. c. Place the patient in a room close to the nurses station. d. Ask the patient why the wandering episodes have occurred.

c. Place the patient in a room close to the nurses station.

When admitting a patient who has just arrived on the unit with a severe headache, what should the nurse do first? a. Complete only basic demographic data before addressing the patients pain. b. Medicate the patient for the headache before doing the health history and examination. c. Take the initial vital signs and then address the headache before completing the health history. d. Inform the patient that the headache will be treated as soon as the health history is completed.

c. Take the initial vital signs and then address the headache before completing the health history.

A 28-year-old patient who has deep human bite wounds on the left hand is being treated in the urgent care center. Which action will the nurse plan to take? a. Prepare to administer rabies immune globulin (BayRab). b. Assist the health care provider with suturing of the bite wounds. c. Teach the patient the reason for the use of prophylactic antibiotics. d. Keep the wounds dry until the health care provider can assess them.

c. Teach the patient the reason for the use of prophylactic antibiotics.

The charge nurse is observing a new staff nurse who is assessing a patient with a traumatic spinal cord injury for sensation. Which action indicates a need for further teaching of the new nurse about neurologic assessment? a. The new nurse tests for light touch before testing for pain. b. The new nurse has the patient close the eyes during testing. c. The new nurse asks the patient if the instrument feels sharp. d. The new nurse uses an irregular pattern to test for intact touch.

c. The new nurse asks the patient if the instrument feels sharp.

A patient is diagnosed with both human immunodeficiency virus (HIV) and active tuberculosis (TB) disease. Which information obtained by the nurse is most important to communicate to the health care provider? a. The Mantoux test had an induration of 7 mm. b. The chest-x-ray showed infiltrates in the lower lobes. c. The patient is being treated with antiretrovirals for HIV infection. d. The patient has a cough that is productive of blood-tinged mucus.

c. The patient is being treated with antiretrovirals for HIV infection.

Which information is most important for the nurse to report to the health care provider before a patient with type 2 diabetes is prepared for a coronary angiogram? a. The patients most recent HbA1C was 6.5%. b. The patients admission blood glucose is 128 mg/dL. c. The patient took the prescribed metformin (Glucophage) today. d. The patient took the prescribed captopril (Capoten) this morning.

c. The patient took the prescribed metformin (Glucophage) today.

hile the nurse is transporting a patient on a stretcher to the radiology department, the patient begins having a tonic-clonic seizure. Which action should the nurse take? a. Insert an oral airway during the seizure to maintain a patent airway. b. Restrain the patients arms and legs to prevent injury during the seizure. c. Time and observe and record the details of the seizure and postictal state. d. Avoid touching the patient to prevent further nervous system stimulation.

c. Time and observe and record the details of the seizure and postictal state.

Following rectal surgery, a patient voids about 50 mL of urine every 30 to 60 minutes for the first 4 hours. Which nursing action is most appropriate? a. Monitor the patients intake and output over night. b. Have the patient drink small amounts of fluid frequently. c. Use an ultrasound scanner to check the postvoiding residual volume. d. Reassure the patient that this is normal after rectal surgery because of anesthesia.

c. Use an ultrasound scanner to check the postvoiding residual volume.

Employee health test results reveal a tuberculosis (TB) skin test of 16-mm induration and a negative chest x-ray for a staff nurse working on the pulmonary unit. The nurse has no symptoms of TB. Which information should the occupational health nurse plan to teach the staff nurse? a. Standard four-drug therapy for TB b. Need for annual repeat TB skin testing c. Use and side effects of isoniazid (INH) d. Bacille Calmette-Gurin (BCG) vaccine

c. Use and side effects of isoniazid (INH)

The nurse assesses a patient with a history of asthma. Which assessment finding indicates that the nurse should take immediate action? a. Pulse oximetry reading of 91% b. Respiratory rate of 26 breaths/minute c. Use of accessory muscles in breathing d. Peak expiratory flow rate of 240 L/minute

c. Use of accessory muscles in breathing (indicated respiratory distress)

Which information from a patients urinalysis requires that the nurse notify the health care provider? a. pH 6.2 b. Trace protein c. WBC 20 to 26/hpf d. Specific gravity 1.021

c. WBC 20 to 26/hpf (The increased number of white blood cells (WBCs) indicates the presence of urinary tract infection or inflammation)

The nurse will anticipate teaching a patient with nephrotic syndrome who develops flank pain about treatment with a. antibiotics. b. antifungals. c. anticoagulants. d. antihypertensives.

c. anticoagulants. (flank pain in nephrotic syndrome suggests renal vein thrombosis)

A patient who has acute glomerulonephritis is hospitalized with hyperkalemia. Which information will the nurse monitor to evaluate the effectiveness of the prescribed calcium gluconate IV? a. Urine volume b. Calcium level c. Cardiac rhythm d. Neurologic status

c. cardiac rhythm

When preparing a female patient with bladder cancer for intravesical chemotherapy, the nurse will teach about a. premedicating to prevent nausea. b. obtaining wigs and scarves to wear. c. emptying the bladder before the medication. d. maintaining oral care during the treatments.

c. emptying the bladder before the medication.

Which nursing diagnosis statement is written correctly? a. altered tissue perfusion related to heart failure b. risk for impaired tissue integrity related to sacral redness c. ineffective coping related to response to biopsy results d. altered urinary elimination related to UTI

c. ineffective coping related to response to biopsy results

When a patients urine dipstick test indicates a small amount of protein, the nurses next action should be to a. send the urine specimen to the lab to test for ketones b. obtain a clean catch for culture & sensitivity c. inquire about which meds the pt is currently taking d. ask the pt about family hx of chronic renal failure

c. inquire about which meds the pt is currently taking (Normally the urinalysis will show zero to trace amounts of protein, but some medications may give false- positive readings.)

While the nurse is transporting a patient on a stretcher to the radiology department, the patient begins having a tonic-clonic seizure. Which action should the nurse take? a. Insert an oral airway during the seizure to maintain a patent airway. b. Restrain the patients arms and legs to prevent injury during the seizure. c. Time and observe and record the details of the seizure and postictal state. d. Avoid touching the patient to prevent further nervous system stimulation.

c. time & observe & record the details of the seizure & the postictal state

A nurse is caring for a client who had an MI 24 hr ago. At this time, it is essential for the nurse to look for which of the following? a. sepsis b. pericarditis c. ventricular dysrhytmias d. pulmonary emboli

c. ventricular dysrhythmias

A coronary care unit (CCU) nurse is caring for a client admitted with acute myocardial infarction (MI). The nurse monitors for which most common complication of MI?

cardiac dysrhythmias

After change-of-shift report, which patient should the nurse assess first? a. 72-year-old with cor pulmonale who has 4+ bilateral edema in his legs and feet b. 28-year-old with a history of a lung transplant and a temperature of 101 F (38.3 C) c. 40-year-old with a pleural effusion who is complaining of severe stabbing chest pain d. 64-year-old with lung cancer and tracheal deviation after subclavian catheter insertion

d. 64-year-old with lung cancer and tracheal deviation after subclavian catheter insertion

The emergency department (ED) triage nurse is assessing four victims involved in a motor vehicle collision. Which patient has the highest priority for treatment? a. A patient with no pedal pulses. b. A patient with an open femur fracture. c. A patient with bleeding facial lacerations. d. A patient with paradoxic chest movements.

d. A patient with paradoxic chest movements.

Which prescribed intervention will the nurse implement first for a patient in the emergency department who is experiencing continuous tonic-clonic seizures? a. Give phenytoin (Dilantin) 100 mg IV. b. Monitor level of consciousness (LOC). c. Obtain computed tomography (CT) scan. d. Administer lorazepam (Ativan) 4 mg IV.

d. Administer lorazepam (Ativan) 4 mg IV.

A patient who has a history of chronic obstructive pulmonary disease (COPD) was hospitalized for increasing shortness of breath and chronic hypoxemia (SaO2 levels of 89% to 90%). In planning for discharge, which action by the nurse will be most effective in improving compliance with discharge teaching? a. Start giving the patient discharge teaching on the day of admission. b. Have the patient repeat the instructions immediately after teaching. c. Accomplish the patient teaching just before the scheduled discharge. d. Arrange for the patients caregiver to be present during the teaching.

d. Arrange for the patients caregiver to be present during the teaching.

The nurse palpates the posterior chest while the patient says 99 and notes absent fremitus. Which action should the nurse take next? a. Palpate the anterior chest and observe for barrel chest. b. Encourage the patient to turn, cough, and deep breathe. c. Review the chest x-ray report for evidence of pneumonia. d. Auscultate anterior and posterior breath sounds bilaterally.

d. Auscultate anterior and posterior breath sounds bilaterally.

A 54-year-old patient arrives in the emergency department (ED) after exposure to powdered lime at work. Which action should the nurse take first? a. Obtain the patients vital signs. b. Obtain a baseline complete blood count. c. Decontaminate the patient by showering with water. d. Brush off any visible powder on the skin and clothing.

d. Brush off any visible powder on the skin and clothing.

The nurse identifies a patient with type 1 diabetes and a history of herpes simplex infection as being at risk for Bells palsy. Which information should the nurse include in teaching the patient? a. You may be able to prevent Bells palsy by doing facial exercises regularly. b. Prophylactic treatment of herpes with antiviral agents prevents Bells palsy. c. Medications to treat Bells palsy work only if started before paralysis onset. d. Call the doctor if you experience pain or develop herpes lesions near the ear.

d. Call the doctor if you experience pain or develop herpes lesions near the ear. (Pain or herpes lesions near the ear may indicate the onset of Bells palsy and rapid corticosteroid treatment may reduce the duration of Bells palsy symptoms.)

A nurse is assigned as a case manager for a hospitalized patient with a spinal cord injury. The patient can expect the nurse functioning in this role to perform which activity? a. Care for the patient during hospitalization for the injuries. b. Assist the patient with home care activities during recovery. c. Determine what medical care the patient needs for optimal rehabilitation. d. Coordinate the services that the patient receives in the hospital and at home.

d. Coordinate the services that the patient receives in the hospital and at home.

The nurse is caring for a 78-year-old patient who was hospitalized 2 days earlier with community-acquired pneumonia. Which assessment information is most important to communicate to the health care provider? a. Scattered crackles bilaterally in the posterior lung bases. b. Persistent cough that is productive of blood-tinged sputum. c. Temperature of 101.5 F (38.6 C) after 2 days of IV antibiotic therapy. d. Decreased oxygen saturation to 90% with 100% O2 by non-rebreather mask.

d. Decreased oxygen saturation to 90% with 100% O2 by non-rebreather mask.

The nurse will anticipate teaching a patient with a possible seizure disorder about which test? a. Cerebral angiography b. Evoked potential studies c. Electromyography (EMG) d. Electroencephalography (EEG)

d. Electroencephalography (EEG)

The nurse is performing tuberculosis (TB) skin tests in a clinic that has many patients who have immigrated to the United States. Which question is most important for the nurse to ask before the skin test? a. Is there any family history of TB? b. How long have you lived in the United States? c. Do you take any over-the-counter (OTC) medications? d. Have you received the bacille Calmette-Gurin (BCG) vaccine for TB?

d. Have you received the bacille Calmette-Gurin (BCG) vaccine for TB?

11. After providing a patient with discharge instructions on the management of a new permanent pacemaker, the nurse knows that teaching has been effective when the patient states a. I will avoid cooking with a microwave oven or being near one in use. b. It will be 1 month before I can take a bath or return to my usual activities. c. I will notify the airlines when I make a reservation that I have a pacemaker. d. I wont lift the arm on the pacemaker side up very high until I see the doctor.

d. I wont lift the arm on the pacemaker side up very high until I see the doctor.

A patient with acute respiratory distress syndrome (ARDS) who is intubated and receiving mechanical ventilation develops a right pneumothorax. Which action will the nurse anticipate taking next? a. Increase the tidal volume and respiratory rate. b. Increase the fraction of inspired oxygen (FIO2). c. Perform endotracheal suctioning more frequently. d. Lower the positive end-expiratory pressure (PEEP).

d. Lower the positive end-expiratory pressure (PEEP).

A patient admitted with acute dyspnea is newly diagnosed with dilated cardiomyopathy. Which information will the nurse plan to teach the patient about managing this disorder? a. A heart transplant should be scheduled as soon as possible. b. Elevating the legs above the heart will help relieve dyspnea. c. Careful compliance with diet and medications will prevent heart failure. d. Notify the doctor about any symptoms of heart failure such as shortness of breath.

d. Notify the doctor about any symptoms of heart failure such as shortness of breath.

A patient hospitalized with chronic obstructive pulmonary disease (COPD) is being discharged home on oxygen therapy. Which instruction should the nurse include in the discharge teaching? a. Storage of oxygen tanks will require adequate space in the home. b. Travel opportunities will be limited because of the use of oxygen. c. Oxygen flow should be increased if the patient has more dyspnea. d. Oxygen use can improve the patients prognosis and quality of life.

d. Oxygen use can improve the patients prognosis and quality of life.

Which of these nursing actions for a 64-year-old patient with Guillain-Barr syndrome is most appropriate for the nurse to delegate to an experienced unlicensed assistive personnel (UAP)? a. Nasogastric tube feeding q4hr b. Artificial tear administration q2hr c. Assessment for bladder distention q2hr d. Passive range of motion to extremities q4hr

d. Passive range of motion to extremities q4hr (Assisting a patient with movement is included in UAP education and scope of practice. Administration of tube feedings, administration of ordered medications, and assessment are skills requiring more education and scope of practice, and the RN should perform these skills.)

Which information about a 72-year-old patient who has a new prescription for phenytoin (Dilantin) indicates that the nurse should consult with the health care provider before administration of the medication? a. Patient has generalized tonic-clonic seizures. b. Patient experiences an aura before seizures. c. Patients most recent blood pressure is 156/92 mm Hg. d. Patient has minor elevations in the liver function tests.

d. Patient has minor elevations in the liver function tests.

After change-of-shift report on the Alzheimers disease/dementia unit, which patient will the nurse assess first? a. Patient who has not had a bowel movement for 5 days b. Patient who has a stage II pressure ulcer on the coccyx c. Patient who is refusing to take the prescribed medications d. Patient who developed a new cough after eating breakfast

d. Patient who developed a new cough after eating breakfast

A patient who was admitted with a myocardial infarction experiences a 45-second episode of ventricular tachycardia, then converts to sinus rhythm with a heart rate of 98 beats/minute. Which of the following actions should the nurse take next? a. Immediately notify the health care provider. b. Document the rhythm and continue to monitor the patient. c. Perform synchronized cardioversion per agency dysrhythmia protocol. d. Prepare to give IV amiodarone (Cordarone) per agency dysrhythmia protocol.

d. Prepare to give IV amiodarone (Cordarone) per agency dysrhythmia protocol.

During the primary survey of a patient with severe leg trauma, the nurse observes that the patients left pedal pulse is absent and the leg is swollen. Which action will the nurse take next? a. Send blood to the lab for a complete blood count. b. Assess further for a cause of the decreased circulation. c. Finish the airway, breathing, circulation, disability survey. d. Start normal saline fluid infusion with a large-bore IV line.

d. Start normal saline fluid infusion with a large-bore IV line.

A 19-year-old is brought to the emergency department (ED) with multiple lacerations and tissue avulsion of the left hand. When asked about tetanus immunization, the patient denies having any previous vaccinations. The nurse will anticipate giving a. tetanus immunoglobulin (TIG) only. b. TIG and tetanus-diphtheria toxoid (Td). c. tetanus-diphtheria toxoid and pertussis vaccine (Tdap) only. d. TIG and tetanus-diphtheria toxoid and pertussis vaccine (Tdap).

d. TIG and tetanus-diphtheria toxoid and pertussis vaccine (Tdap).

A patient with chronic obstructive pulmonary disease (COPD) has a nursing diagnosis of impaired breathing pattern related to anxiety. Which nursing action is most appropriate to include in the plan of care? a. Titrate oxygen to keep saturation at least 90%. b. Discuss a high-protein, high-calorie diet with the patient. c. Suggest the use of over-the-counter sedative medications. d. Teach the patient how to effectively use pursed lip breathing.

d. Teach the patient how to effectively use pursed lip breathing.

The nurse teaches a patient who has asthma about peak flow meter use. Which action by the patient indicates that teaching was successful? a. The patient inhales rapidly through the peak flow meter mouthpiece. b. The patient takes montelukast (Singulair) for peak flows in the red zone. c. The patient calls the health care provider when the peak flow is in the green zone. d. The patient uses albuterol (Proventil) metered dose inhaler (MDI) for peak flows in the yellow zone.

d. The patient uses albuterol (Proventil) metered dose inhaler (MDI) for peak flows in the yellow zone.

The nurse is obtaining a health history from a 24-year-old patient with hypertrophic cardiomyopathy. Which information obtained by the nurse is most important? a. The patient has a history of a recent upper respiratory infection. b. The patient has a family history of coronary artery disease (CAD). c. The patient reports using cocaine "a couple of times" as a teenager. d. The patient's 29-year-old brother died from a sudden cardiac arrest.

d. The patient's 29-year-old brother died from a sudden cardiac arrest.

A 62-year-old patient who has Parkinsons disease is taking bromocriptine (Parlodel). Which information obtained by the nurse may indicate a need for a decrease in the dose? a. The patient has a chronic dry cough. b. The patient has four loose stools in a day. c. The patient develops a deep vein thrombosis. d. The patients blood pressure is 92/52 mm Hg.

d. The patients blood pressure is 92/52 mm Hg. (hypoTN is a/e)

The nurse completes an admission assessment on a patient with asthma. Which information given by patient is most indicative of a need for a change in therapy? a. The patient uses albuterol (Proventil) before any aerobic exercise. b. The patient says that the asthma symptoms are worse every spring. c. The patients heart rate increases after using the albuterol (Proventil) inhaler. d. The patients only medications are albuterol (Proventil) and salmeterol (Serevent).

d. The patients only medications are albuterol (Proventil) and salmeterol (Serevent).

A patient with chronic obstructive pulmonary disease (COPD) arrives in the emergency department complaining of shortness of breath and dyspnea on minimal exertion. Which assessment finding by the nurse is most important to report to the health care provider? a. The patient has bibasilar lung crackles. b. The patient is sitting in the tripod position. c. The patients respirations have decreased from 30 to 10 breaths/minute. d. The patients pulse oximetry indicates an O2 saturation of 91%.

d. The patients pulse oximetry indicates an O2 saturation of 91%.

Four hours after mechanical ventilation is initiated for a patient with chronic obstructive pulmonary disease (COPD), the patients arterial blood gas (ABG) results include a pH of 7.51, PaO2 of 82 mm Hg, PaCO2 of 26 mm Hg, and HCO3 of 23 mEq/L (23 mmol/L). The nurse will anticipate the need to a. increase the FIO2. b. increase the tidal volume. c. increase the respiratory rate. d. decrease the respiratory rate.

d. decrease the respiratory rate. (respiratory alkalosis d/t inc RR)

Nadolol (Corgard) is prescribed for a patient with chronic stable angina and left ventricular dysfunction. To determine whether the drug is effective, the nurse will monitor for a. decreased blood pressure and heart rate. b. fewer complaints of having cold hands and feet. c. improvement in the strength of the distal pulses. d. the ability to do daily activities without chest pain.

d. the ability to do daily activities without chest pain.

A 79-year-old man has been admitted with benign prostatic hyperplasia. What is most appropriate to include in the nursing plan of care?

pad the pt's bed to accommodate overflow incontinence

Which medication information will the nurse identify as a concern for a patients musculoskeletal status?

pt has severe asthma & req tx w/ PO corticosteroids

A patient who was involved in a motor vehicle crash has had a tracheostomy placed to allow for continued mechanical ventilation. How should the nurse interpret the following arterial blood gas results: pH 7.48, PaO2 85 mm Hg, PaCO2 32 mm Hg, and HCO3 25 mEq/L?

respiratory alkalosis

A 73-year-old patient with Parkinsons disease has a nursing diagnosis of impaired physical mobility related to bradykinesia. Which action will the nurse include in the plan of care?

suggest that the pt rock from side to side to initiate leg movement

Which information will the nurse include in the asthma teaching plan for a patient being discharged?

tremors are expected s/e of rapid acting broncho dilators (SABAs)


Related study sets

3. A3 Unit 5 Chapter 62: Management of Patients with Cerebrovascular Disorders (Study)

View Set

world geography chapter 24 and 25 review

View Set

Chapter 6 "The American Revolution"

View Set

Chapter 50: Assessment and Management of Patients With Biliary Disorders CN3

View Set

Microbiology Final Exam (all cards)

View Set